Levi-Civita proofs for divergence of curls, etc

In summary, the conversation discusses the process of proving equations using Levi-Civita notation and its application in physics. The participants share their attempts at proving given equations and ask for feedback. They eventually come to a conclusion and thank each other for their help.
  • #1
theuserman
11
0
I've also posted this in the Physics forum as it applies to some physical aspects as well.
---

I want to know if I'm on the right track here. I'm asked to prove the following.

a) [tex]\nabla \cdot (\vec{A} \times \vec{B}) = \vec{B} \cdot (\nabla \times \vec{A}) - \vec{A} \cdot (\nabla \times \vec{B})[/tex]
b) [tex]\nabla \times (f \vec{A}) = f(\nabla \times \vec{A}) - \vec{A} \times (\nabla f)[/tex] (where f is a scalar function)

And want (read: need to, due to a professor's insistence) to prove these using Levi-Civita notation. I've used the following for reference:
http://www.uoguelph.ca/~thopman/246/indicial.pdf and http://folk.uio.no/patricg/teaching/a112/levi-civita/

Here's my attempts - I need to see if I have this notation down correctly...

a) [tex]\nabla \cdot (\vec{A} \times \vec{B}) [/tex]

= [tex]\partial_i \hat{u}_i \cdot \epsilon_{jkl} \vec{A}_j \vec{B}_k \hat{u}_l[/tex]

= [tex]\partial_i \vec{A}_j \vec{B}_k \hat{u}_i \cdot \hat{u}_l \epsilon_{jkl}[/tex]

Now I thought it'd be wise to use the identity that [tex]\hat{u}_i \cdot \hat{u}_l = \delta_{il}[/tex].

= [tex]\partial_i \vec{A}_j \vec{B}_k \delta_{il} \epsilon_{jkl}[/tex]

In which we make i = l (and the [tex]\delta_{il}[/tex] goes to 1).

= [tex]\partial_i \vec{A}_j \vec{B}_k \epsilon_{jki} [/tex]

Then using 'scalar derivative product rules' we get two terms. Now, here's where I get a little mixed up. I'm wondering if we rearrange the terms and then modify the epsilon to go in order the the terms.

= [tex] \vec{B}_k \partial_i \vec{A}_j \epsilon_{kij} + \vec{A}_j \partial_i \vec{B}_k \epsilon_{jik}[/tex]

Now since the first epsilon is 'even' it remains positive, the other epsilon is 'odd' so that term becomes negative and we end up with the required result.

= [tex] \vec{B} (\nabla \times \vec{A}) - \vec{A} (\nabla \times \vec{B}) [/tex]

b) [tex]\nabla \times (f \vec{A}) [/tex] (where f is a scalar function)

= [tex] \partial_i f \vec{A}_j \hat{u}_k \epsilon_{ijk}[/tex]

= [tex]f \partial_i \vec{A}_j \hat{u}_k \epsilon_{ijk}+ \vec{A}_j \partial_i f \hat{u}_k \epsilon_{jik}[/tex]

Once again, the first epsilon is the positive ('even') while the other is negative ('odd').

= [tex] f (\nabla \times \vec{A}) - \vec{A}(\nabla f)[/tex]

Man, my hands hurt from all that tex work :P Been awhile for me.
Since my teacher refuses to tell me if this is the correct method (he's only willing to show the concepts, and while I can appreciate that I don't want my mark to go to hell), can anyone help me out?
 
Last edited by a moderator:
Mathematics news on Phys.org
  • #2
Some new equations I need to prove and have no idea how to go about it... I'll show what I've attempted so far.

a) [tex]\nabla (\vec{A} \cdot \vec{B}) = \vec{A} \times (\nabla \times \vec{B}) + \vec{B} \times (\nabla \times \vec{A}) + (\vec{A} \cdot \nabla)\vec{B} + (\vec{B} \cdot \nabla) \vec{A} [/tex]

b) [tex]\nabla \times (\vec{A} \times \vec{B}) = (\vec{B} \cdot \nabla)\vec{A} - (\vec{A} \cdot \nabla)\vec{B} + \vec{A}(\nabla \cdot \vec{B}) - \vec{B}(\nabla \cdot \vec{A})[/tex]

Attempts:

a) [tex]\nabla (\vec{A} \cdot \vec{B}) [/tex]

= [tex] \delta_i (\vec{A}_j \hat{u}_j \cdot \vec{B}_k \hat{u}_k) [/tex]

= [tex] \delta_i (\vec{A}_j \vec{B}_j) [/tex]

= [tex] \vec{B}_j \delta_i \vec{A}_j + \vec{A}_j \delta_i \vec{B}_j [/tex]

Now I'm thinking we do the other side of the equation and see if they add up??
1st term :
[tex]\vec{A} \times (\nabla \times \vec{B})[/tex]

= [tex] \epsilon_{ijk} \vec{A}_j (\nabla \times \vec{B})_k [/tex]

= [tex] \epsilon_{ijk} \vec{A}_j \epsilon_{klm} \nabla_l \vec{B}_m [/tex]

= [tex] \epsilon_{kij} \epsilon_{klm} \vec{A}_j \nabla_l \vec{B}_m [/tex]

= [tex] (\delta_{il} \delta_{jm} - \delta_{im} \delta_{jl}) \vec{A}_j \nabla_l \vec{B}_m [/tex]

So we make i=l, j=m and for the other case i=m and j=l

= [tex] \vec{A}_j \nabla_l \vec{B}_j - \vec{A}_l \nabla_l \vec{B}_m [/tex]

= [tex] \vec{A}_j \partial_l \vec{B}_j - \vec{A}_l \partial_l \vec{B}_m [/tex]

Which seems to resemble what I got when I did this normally...

2nd term : (I'm guessing we just switch the terms)
[tex]\vec{B} \times (\nabla \times \vec{A}) [/tex]

= [tex] \vec{B}_j \partial_l \vec{A}_j - \vec{B}_l \partial_l \vec{A}_m [/tex]

3rd term:
[tex] (\vec{A} \cdot \nabla)\vec{B} [/tex]

= [tex](\vec{A}_i \cdot \nabla_j)\vec{B}_k [/tex]

= [tex](\vec{A}_i \nabla_i) \vec{B}_k [/tex]

= [tex]\vec{A}_i \nabla_i \vec{B}_k [/tex]

4th term:
[tex] (\vec{B} \cdot \nabla)\vec{A} [/tex]

= [tex]\vec{B}_i \nabla_i \vec{A}_k [/tex]

Adding them all up gets us:

[tex] \vec{A}_j \partial_l \vec{B}_j - \vec{A}_l \partial_l \vec{B}_m + \vec{B}_j \partial_l \vec{A}_j - \vec{B}_l \partial_l \vec{A}_m + \vec{A}_i \partial_i \vec{B}_k + \vec{B}_i \partial_i \vec{A}_k[/tex]

Since most of these are dummy variables I can change them to...[tex] \vec{A}_j \partial_l \vec{B}_j - \vec{A}_l \partial_l \vec{B}_m + \vec{B}_j \partial_l \vec{A}_j - \vec{B}_l \partial_l \vec{A}_m + \vec{A}_l \partial_l \vec{B}_m + \vec{B}_l \partial_l \vec{A}_m[/tex]

Which is [tex] \vec{B}_j \delta_i \vec{A}_j + \vec{A}_j \delta_i \vec{B}_j [/tex] !
HAH!

b) [tex]\nabla \times (\vec{A} \times \vec{B}) [/tex] I'm pretty sure I have this down.

= [tex]\partial_l \hat{u}_l \times (\vec{A}_i \vec{B}_j \hat{u}_k \epsilon_{ijk}) [/tex]

=[tex]\partial_l \vec{A}_i \vec{B}_j \epsilon_{ijk} (\hat{u}_l \times \hat{u}_k) [/tex]

[[tex](\hat{u}_l \times \hat{u}_k) = \hat{u}_m \epsilon_{lkm}[/tex]]

=[tex]\partial_l \vec{A}_i \vec{B}_j \hat{u}_m \epsilon_{ijk} \epsilon_{mlk} [/tex]

=[tex]\partial_l \vec{A}_i \vec{B}_j \hat{u}_m (\delta_{im} \delta_{jl} - \delta_{il} \delta_{jm}) [/tex]

=[tex]\partial_j \vec{A}_i \vec{B}_j \hat{u}_i- \partial_i \vec{A}_i \vec{B}_j \hat{u}_j [/tex]

Using scalar derivative scalar product rule:

= [tex]\vec{A}_i \partial_j \vec{B}_j \hat{u}_i + \partial_j \vec{B}_j \vec{A}_i \hat{u}_i - (\vec{A}_i \partial_i \vec{B}_j \hat{u}_j + \vec{B}_j \vec{A}_i \partial_i \hat{u}_j) [/tex]

= [tex]\vec{A}(\nabla \cdot \vec{B}) + (\vec{B} \cdot \nabla)\vec{A} - (\vec{A} \cdot \nabla)\vec{B} - \vec{B}(\nabla \cdot \vec{A}) [/tex]

Alright, so it took me 2 hours to type it but I managed to figure it out.
Thanks anyway.
 
  • #3
theuserman said:
I've also posted this in the Physics forum as it applies to some physical aspects as well.
---

I want to know if I'm on the right track here. I'm asked to prove the following.

a) [tex]\nabla \cdot (\vec{A} \times \vec{B}) = \vec{B} \cdot (\nabla \times \vec{A}) - \vec{A} \cdot (\nabla \times \vec{B})[/tex]
...
a) [tex]\nabla \cdot (\vec{A} \times \vec{B}) [/tex]

= [tex]\partial_i \hat{u}_i \cdot \epsilon_{jkl} \vec{A}_j \vec{B}_k \hat{u}_l[/tex]

= [tex]\partial_i \vec{A}_j \vec{B}_k \hat{u}_i \cdot \hat{u}_l \epsilon_{jkl}[/tex]

Now I thought it'd be wise to use the identity that [tex]\hat{u}_i \cdot \hat{u}_l = \delta_{il}[/tex].

I haven't got a lot of wherewithall at the moment to put a lot into this, but I can help a little with equation a)

Change your equation to:

a) [tex]\nabla \cdot (\vec{A} \times \vec{B}) = \partial_i \hat{u}_i \cdot \epsilon_{jki} \vec{A}_j \vec{B}_k \hat{u}_i[/tex]

Then using 'scalar derivative product rules' we get two terms. Now, here's where I get a little mixed up. I'm wondering if we rearrange the terms and then modify the epsilon to go in order the the terms.

= [tex] \vec{B}_k \partial_i \vec{A}_j \epsilon_{kij} + \vec{A}_j \partial_i \vec{B}_k \epsilon_{jik}[/tex]

Now since the first epsilon is 'even' it remains positive, the other epsilon is 'odd' so that term becomes negative and we end up with the required result.

= [tex] \vec{B} (\nabla \times \vec{A}) - \vec{A} (\nabla \times \vec{B}) [/tex]

For my own work, when things get a little dicy, I define e.

[tex]\ \ e_{ijk} = 1[/tex] where i,j,k are odd (cyclic) permutations of (1,2,3) and otherwise the permutations are are 0.

[tex]\ e_{123} = e_{231} = e_{312} = 1[/tex]

To make things a bit transparent, define [tex]\ \overline{e}[/tex]

[tex]\ \overline{e}_{ijk} = -1[/tex] where i,j,k are odd permutations of (1,2,3) and otherwise are 0.

[tex]\ \overline{e}_{321} = \overline{e}_{213} = \overline{e}_{132} = -1 [/tex]

So you get

[tex]\epsilon_{ijk} = e_{ijk} + \overline{e}_{ijk}[/tex]
 
  • #4
Phrak,
Thanks for the tip with [tex] \epsilon [/tex], as for your first suggestion it makes sense to make it i... Since it is a dummy variable.

Thanks.
 
  • #5
Just completeness, I'm going to prove that:

[tex]
\nabla \times (\nabla \times \vec{v}) = \nabla(\nabla \cdot \vec{v}) - \nabla^2\vec{v}
[/tex]

So we have
[tex]
\nabla \times (\nabla \times \vec{v})
[/tex]

[tex]
= \partial_i(\nabla \times \vec{v})_j \epsilon_{ijk}
[/tex]

[tex]
= \partial_i(\partial_l v_m \epsilon_{jlm})_j \epsilon_{ijk}
[/tex]

[tex]
=\partial_i \partial_l v_m \epsilon_{jlm} \epsilon_{ijk}
[/tex]

[tex]
= \partial_i \partial_l v_m (\delta_{lk}\delta_{mi} - \delta_{li}\delta_{mk})
[/tex]

[tex]
= \delta_{lk}\delta_{mi} \partial_i \partial_l v_m - \delta_{li}\delta_{mk}\partial_i \partial_l v_m
[/tex]

[tex]
= \partial_m \partial_l v_m - \partial_i \partial_i v_m
[/tex]

[tex]
= \partial_l \partial_m v_m - \partial_i^2 v_m
[/tex]

[tex]
= \nabla(\nabla \cdot \vec{v}) - \nabla^2 \vec{v}
[/tex]

Viola.
 
  • #6
theuserman said:
Just completeness, I'm going to prove that:

[tex]
\nabla \times (\nabla \times \vec{v}) = \nabla(\nabla \cdot \vec{v}) - \nabla^2\vec{v}
[/tex]

So we have
[tex]
\nabla \times (\nabla \times \vec{v})
[/tex]

[tex]
= \partial_i(\nabla \times \vec{v})_j \epsilon_{ijk}
[/tex]

[tex]
= \partial_i(\partial_l v_m \epsilon_{jlm})_j \epsilon_{ijk}
[/tex]

[tex]
=\partial_i \partial_l v_m \epsilon_{jlm} \epsilon_{ijk}
[/tex]

[tex]
= \partial_i \partial_l v_m (\delta_{lk}\delta_{mi} - \delta_{li}\delta_{mk})
[/tex]

[tex]
= \delta_{lk}\delta_{mi} \partial_i \partial_l v_m - \delta_{li}\delta_{mk}\partial_i \partial_l v_m
[/tex]

[tex]
= \partial_m \partial_l v_m - \partial_i \partial_i v_m
[/tex]

[tex]
= \partial_l \partial_m v_m - \partial_i^2 v_m
[/tex]

[tex]
= \nabla(\nabla \cdot \vec{v}) - \nabla^2 \vec{v}
[/tex]

Viola.

i am wondering, why it is such from the last 4th step onwards

going from the last 4th to last 3rd step, why does the variables become as such? how exactly does one compute the kronecker delta here?

also on the last step, del(del.v)
does it mean that the del(del is a multiplication? it is neither cross nor dot product? so i should treat it as a scalar multiplication?

also, del del v are all vectors right? so a dot product gives a number, and the outter del is just a scalar multiplcation?

also, when we take cross product , say AXB
where [AXB]i = EijkAjBk
when we say the i-th component is as such, does it mean the unit vector i, which is the x-plane, or is i just a dummy variable?
also what does it mean by the i-th component? does it mean there's also a j and k? and when we sum them up we get the vector [AXB] ?
 
Last edited:

1. What is a Levi-Civita proof for divergence of curls?

A Levi-Civita proof for divergence of curls is a mathematical method used to prove the divergence of a curl field, which is a vector field in three-dimensional space. It involves using the properties of the Levi-Civita symbol, which represents the permutation of indices in three dimensions, to simplify the proof.

2. Why is a Levi-Civita proof important in science?

A Levi-Civita proof is important in science because it provides a rigorous and systematic way to prove the divergence of a vector field. This is useful in many areas of science, such as physics and engineering, where vector fields are commonly used to describe physical phenomena.

3. How do you apply Levi-Civita proofs to prove divergence of curls?

To apply a Levi-Civita proof, you first need to express the vector field in terms of its components. Then, you use the properties of the Levi-Civita symbol to simplify the expression. By rearranging and manipulating the expression, you can eventually show that the divergence of the curl is equal to zero, which proves the divergence of the vector field.

4. Are there any limitations to using Levi-Civita proofs for divergence of curls?

One limitation of using Levi-Civita proofs is that they can only be applied to vector fields in three-dimensional space. Additionally, they may not be the most efficient method for proving divergence of curls in some cases, as there may be alternative methods that are more straightforward.

5. How can I learn more about Levi-Civita proofs for divergence of curls?

There are many resources available for learning about Levi-Civita proofs and their applications. You can find textbooks, online tutorials, and videos that explain the concept in depth. It may also be helpful to consult with a mathematics or physics professor for further guidance and clarification.

Similar threads

  • Calculus and Beyond Homework Help
Replies
5
Views
954
Replies
4
Views
819
  • Calculus and Beyond Homework Help
Replies
9
Views
762
  • Classical Physics
Replies
1
Views
628
  • Quantum Physics
Replies
5
Views
528
  • Calculus and Beyond Homework Help
Replies
3
Views
1K
Replies
3
Views
904
  • General Math
Replies
5
Views
2K
  • Calculus and Beyond Homework Help
Replies
2
Views
3K
Back
Top